Re: [obm-l] Qual resposta

2004-02-04 Por tôpico Vitor Paizam
Estive conversando com o Carlos pelo msn e descobri que na verdade a fundação C.CHAGAS acertou na questw e que o problema e muito mais simples do que estava imaginando. Na questw, estava escrito que os multiplos de 4,6 e 12, estavam dentro do conjunto X, logo por exemplo, nw poderiamos ter

Re: [obm-l] Qual resposta

2004-02-04 Por tôpico Augusto Cesar de Oliveira Morgado
Mas nao estava escrito que APENAS esses... == Mensagem enviada pelo CIP WebMAIL - Nova Geração - v. 2.1 CentroIn Internet Provider http://www.centroin.com.br Tel: (21) 2542-4849, (21) 2295-3331Fax: (21) 2295-2978

Re: [obm-l] Qual resposta

2004-02-04 Por tôpico Carlos Alberto
Entendo a frustração do Morgado perante ao exercício... Mas a discussão vai longe. Ao meu parecer o exercício é claro, o problema todo estava na interpretação do mesmo. Pelo que entendi o Morgado está querendo dizer, que o exercício deixa vago a quantidade de elementos, levando a interpretaçao

Re: [obm-l] irracionais

2004-02-04 Por tôpico Artur Costa Steiner
Entao quando decompormos N e verificarmos que as potencias dos primos de N forem multiplos de n a potencia N^(1/n) eh racional ? Eh isso ? Sim, de fato. Neste caso, existe um natural k tal que k^n = N. N^(1/n) neste caso eh nao apenas racional mas inteiro. Se, entrtanto, houver um unico primo p

Re: [obm-l] Parece mas nao eh

2004-02-04 Por tôpico Artur Steiner
Oi salvador, Eu pensei um pouco sobre este problema, mas a unica conclusao a que eu ateh agora cheguei foi a mesma que o Claudio jah apresentou em uma outra mensagem. Sabemos que, se comecarmos com um x(1) 100, para algum n acabaremos tendo necessariamente que x(n) 100. Logo, para analisarmos o

[obm-l] Logaritmos (IEZZI) !!!

2004-02-04 Por tôpico Carlos Alberto
Demonstre que a relação entre os logaritmos de dois números positivos e diferentes de 1 independe da base considerada. [ ] sYahoo! Mail - 6MB, anti-spam e antivírus gratuito. Crie sua conta agora!

[obm-l] Re: [obm-l] Re: [obm-l] PEQUENA DÚVIDA!

2004-02-04 Por tôpico Nicolau C. Saldanha
On Tue, Feb 03, 2004 at 09:25:17PM -0500, [EMAIL PROTECTED] wrote: Para Nicolau ou quem souber ! Por que A probabilidade de o total ser 10 é 3/36 ? Cada possibilidade dentro da lista abaixo tem prob 1/36: (1,1), (1,2), (1,3), (1,4), (1,5), (1,6) (2,1), (2,2), (2,3), (2,4), (2,5), (2,6) ...

Re: [obm-l] Duvida - triangulo

2004-02-04 Por tôpico Paulo Santa Rita
Ola Fael e demais colegas desta lista ... OBM-L, A'B' = CA'' + CB'' quando o triangulo ABC e isosceles, isto e, quando CA=CB. Note que o triangulo A'B'C' e congruente a ABC, mas nao e, necessariamente, a imagem de ABC atraves do espelho que seria a reta s. Para ver isso claramente considere o

[obm-l] problema de Analise

2004-02-04 Por tôpico Artur Costa Steiner
A seguinte conclusao eh interessante e eh tambem interesante de se demonstrar: Sejam f e g funcoes de (0 , inf) em R tais que lim (x - inf) f(x) = inf e lim (x - inf) g(x) = inf. Se existir algum a0 tal que f/g seja limitada em (a, inf), entao lim (x- inf) Ln(f(x))/Ln(g(x)) = 1. Artur

Re: [obm-l] irracionais

2004-02-04 Por tôpico Jefferson Franca
Valeu Domingos eu achei animal a solução"Domingos Jr." [EMAIL PROTECTED] wrote: suponha sqrt(15) + sqrt(10) - sqrt(6) - 3 racional;então (1) = sqrt(15) + sqrt(10) - sqrt(6) é racional =(1)² é racional =15 + 10 + 6 + 2sqrt(150) - 2sqrt(60) - 2sqrt(90) é racional =sqrt(150) - sqrt(60) - sqrt(90) é

[obm-l] Ajude a verificar a conjectura de Goldbach

2004-02-04 Por tôpico Nicolau C. Saldanha
Acabo de descobrir que um português chamado Tomás Oliveira e Silva está fazendo uma grande verificação da conjectura de Goldbach pela internet. Verificar aqui significa aumentar o valor de N para o qual a frase todo número par n, 2 n N, pode ser escrito como uma soma de dois primos já foi

[obm-l] irracionais

2004-02-04 Por tôpico Jefferson Franca
Será q alguém pode indicar algum livro ou nota de aula que esteja na net sobre irracionais e racionais?Yahoo! GeoCities: 15MB de espaço grátis para criar seu web site!

Re: [obm-l] irracionais-livro

2004-02-04 Por tôpico Bruno Lima
cara, tem um livro, acho que e' de Djairo Guedes e se chama Numeros Irracionais e Trancendentes, nao deve ter na internet mas acho que o impa vende e te envia, e deve ficar barato.Jefferson Franca [EMAIL PROTECTED] wrote: Será q alguém pode indicar algum livro ou nota de aula que esteja na net

Re: [obm-l] Qual resposta

2004-02-04 Por tôpico Faelccmm
Pois eu tenho um fasciculo (cheio de erros por sinal :-) e nele ha varias questoes de vestibulares, inclusive esta. Em uma mensagem de 4/2/2004 08:08:47 Hor. de verão leste da Am. Sul, [EMAIL PROTECTED] escreveu: Estive conversando com o Carlos pelo msn e descobri que na verdade a fundação

Re: [obm-l] Duvida - triangulo

2004-02-04 Por tôpico Faelccmm
Ok, Paulo... Agora entendi o por quê da imagem nao ser especular !!! Eu acabei provando isso aqui esbocando um triangulo escaleno, por exemplo, e transladando o segmento A"B" para formar um triangulo retangulo com A`B`, onde A`B` sera a hipotenusa e consequentemente maior que o cateto (A"B" =

Re: [obm-l] Duvida - triangulo

2004-02-04 Por tôpico Paulo Santa Rita
Ola Fael e demais colegas desta lista ... OBM-L, Nao nessariamente. Pode suceder que nenhum lado de A'B'C seja perpendicular a reta s, que e o que ocorre num triangulo acutangulo. Um Abraco Paulo Santa Rita 4,2018,040204 From: [EMAIL PROTECTED] Reply-To: [EMAIL PROTECTED] To: [EMAIL PROTECTED]

Re:[obm-l] problema de Analise

2004-02-04 Por tôpico claudio.buffara
De: [EMAIL PROTECTED] Para: [EMAIL PROTECTED] Cópia: Data: Wed, 4 Feb 2004 14:55:12 -0200 Assunto: [obm-l] problema de Analise A seguinte conclusao eh interessante e eh tambem interesante de se demonstrar: Sejam f e g

Re: [obm-l] irracionais-livro

2004-02-04 Por tôpico Jefferson Franca
valeu brunoBruno Lima [EMAIL PROTECTED] wrote: cara, tem um livro, acho que e' de Djairo Guedes e se chama Numeros Irracionais e Trancendentes, nao deve ter na internet mas acho que o impa vende e te envia, e deve ficar barato.Jefferson Franca [EMAIL PROTECTED] wrote: Será q alguém pode indicar

RE: [obm-l] problema de Analise

2004-02-04 Por tôpico Artur Costa Steiner
Exatamente, Claudio. Voce automaticamente assumiu uma condicao adicional que eu esqueci de mencionar (esqueci mesmo, nao foi proposital nao) e que eh necessaria para que o teorema seja valido: para algum b0, devemos ter w =infimo {f(x)/g(x): x=b} 0. Uma outra prova, aparentemente diferente da

[obm-l] f(x) e f'(x)

2004-02-04 Por tôpico Marcelo Souza
Suponha pum polinomio de quinto grau em x. Como demonstro que se toda raiz de p(x) é real, entaump'(x) tem 4 raizes reias (e p''(x)tem 3 raizes reais...) []'s, M.MSN Messenger: converse com os seus amigos online. Instale grátis. Clique aqui.

Re: [obm-l] f(x) e f'(x)

2004-02-04 Por tôpico Eduardo Casagrande Stabel
Oi Marcelo. Sejam r s duas raízes reais consecutivas do polinômio P(x). No intervalo (r, s) o polinômio assume valoressó positivos ou só negativos. No primeiro caso existe um ponto de máximo localr T s pois P é contínuo no compacto [r, s] e nulo nos extremos, sendo positivo no interior.

Re: [obm-l] f(x) e f'(x)

2004-02-04 Por tôpico Rafael
Marcelo, Pelo que pude entender do problema, você está querendo demonstrar um caso particular do teorema das raízes múltiplas.Vou explicá-lo. Onúmero r,que pertence a C, é raiz de multiplicidade m, que pertence a N*, da equação F(x)=0 se, e somente se, F(x)=(x-r)^m*Q(x) e Q(r)0. Se